Difference between revisions of "2013 AMC 8 Problems/Problem 15"

(Solution)
(Solution)
Line 8: Line 8:
 
This can be brute-forced.
 
This can be brute-forced.
  
<math>90-81=9=3^2</math>,  
+
<math>90-81=9=3^2</math>, so <math>p=2</math>.
  
<math>76-44=32=2^5</math>, and
+
<math>76-44=32=2^5</math>, so  <math>r=5</math>. 
  
<math>1421-125=1296</math>.
+
Additionally, <math>1421-125=1296</math>.
  
 
So <math>p=2</math> and <math>r=5</math>. Then, we find <math>s</math>.
 
So <math>p=2</math> and <math>r=5</math>. Then, we find <math>s</math>.

Revision as of 23:51, 27 November 2013

Problem

If $3^p + 3^4 = 90$, $2^r + 44 = 76$, and $5^3 + 6^s = 1421$, what is the product of $p$, $r$, and $s$?

$\textbf{(A)}\ 27 \qquad \textbf{(B)}\ 40 \qquad \textbf{(C)}\ 50 \qquad \textbf{(D)}\ 70 \qquad \textbf{(E)}\ 90$

Solution

This can be brute-forced.

$90-81=9=3^2$, so $p=2$.

$76-44=32=2^5$, so $r=5$.

Additionally, $1421-125=1296$.

So $p=2$ and $r=5$. Then, we find $s$.

$6*6=36$

$6*36=216$

$216*6=1296=6^4$.

It may help to memorize that $1296$ is $6^4$.

Therefore the answer is $2*5*4=\boxed{\textbf{(B)}\ 40}$.

See Also

2013 AMC 8 (ProblemsAnswer KeyResources)
Preceded by
Problem 14
Followed by
Problem 16
1 2 3 4 5 6 7 8 9 10 11 12 13 14 15 16 17 18 19 20 21 22 23 24 25
All AJHSME/AMC 8 Problems and Solutions

The problems on this page are copyrighted by the Mathematical Association of America's American Mathematics Competitions. AMC logo.png